Maths-cours

COURS & EXERCICES DE MATHÉMATIQUES

Close

Suites arithmético-géométrique - Bac ES/L Amérique du Nord 2013

Exercice 3  (5 points)

Candidats de la série ES n'ayant pas suivi l'enseignement de spécialité et candidats de L.

La bibliothèque municipale étant devenue trop petite, une commune a décidé d'ouvrir une médiathèque qui pourra contenir 100 000 ouvrages au total.

Pour l'ouverture prévue le 1er janvier 2013, la médiathèque dispose du stock de 35 000 ouvrages de l'ancienne bibliothèque augmenté de 7 000 ouvrages supplémentaires neufs offerts par la commune.

Partie A

Chaque année, la bibliothécaire est chargée de supprimer 5% des ouvrages, trop vieux ou abîmés, et d'acheter 6 000 ouvrages neufs.

On appelle unu_{n} le nombre, en milliers, d'ouvrages disponibles le 1er janvier de l'année (2013+n)\left(2013+n\right).

On donne u0=42u_{0}=42.

  1. Justifier que, pour tout entier naturel nn , on a un+1=un×0,95+6u_{n+1}=u_{n}\times 0,95+6.

  2. On propose, ci-dessous, un algorithme, en langage naturel.

    Expliquer ce que permet de calculer cet algorithme.

    Variables :
    \quad U,NU, N
    Initialisation :
    \quad Mettre 4242 dans UU
    \quad Mettre 00 dans NN
    Traitement :
    \quad Tant que U<100U < 100
    \quad \quad UU prend la valeur U×0,95+6U\times 0,95+6
    \quad \quad NN prend la valeur N+1N+1
    \quad Fin du Tant que
    Sortie :
    \quad Afficher NN

  3. À l'aide de votre calculatrice, déterminer le résultat obtenu grâce à cet algorithme.

Partie B

La commune doit finalement revoir ses dépenses à la baisse, elle ne pourra financer que 4 000 nouveaux ouvrages par an au lieu des 6 000 prévus.

On appelle vnv_{n} le nombre, en milliers, d'ouvrages disponibles le 1er janvier de l'année (2013+n)\left(2013 +n\right).

  1. Identifier et écrire la ligne qu'il faut modifier dans l'algorithme pour prendre en compte ce changement.

  2. On admet que vn+1=vn×0,95+4v_{n+1}=v_{n}\times 0,95+4 avec v0=42v_{0}=42.

    On considère la suite (wn)\left(w_{n}\right) définie, pour tout entier nn, par wn=vn80w_{n}=v_{n} - 80.

    Montrer que (wn)\left(w_{n}\right) est une suite géométrique de raison q=0,95q=0,95 et préciser son premier terme w0w_{0}.

  3. On admet que, pour tout entier naturel n:wn=38×(0,95)nn : w_{n}= - 38\times \left(0,95\right)^{n}.

    1. Déterminer la limite de (wn)\left(w_{n}\right).

    2. En déduire la limite de (vn)\left(v_{n}\right).

    3. Interpréter ce résultat.

Corrigé

Partie A

  1. Chaque année 5% des ouvrages sont supprimés. Le nombre d'ouvrages restant est alors :

    (15100)×un=0,95×un\left(1 - \frac{5}{100}\right) \times u_{n} = 0,95\times u_{n} .

    Ensuite, 6 000 ouvrages neufs (soit 6 milliers) sont achetés.

    Le nombre d'ouvrages disponibles le 1er janvier de l'année (2013+n+1)\left(2013+n+1\right) est donc :

    un+1=0,95×un+6u_{n+1}=0,95\times u_{n} +6

  2. Cet algorithme affiche le plus petit rang NN à partir duquel uN>100u_{N} > 100 (c'est à dire à partir duquel le nombre d'ouvrages dépassera 10 000).

  3. A la calculatrice on trouve :

    u2699,45u_{26} \approx 99,45 et u27100,47u_{27} \approx 100,47

    donc le résultat affiché par l'algorithme est 27.

Partie B

  1. Il faut remplacer 6 par 4 à la ligne :

    UU prend la valeur U×0,95+6U\times 0,95+6

    L'algorithme devient :

    Variables :
    \quad U,NU, N
    Initialisation :
    \quad Mettre 4242 dans UU
    \quad Mettre 00 dans NN
    Traitement :
    \quad Tant que U<100U < 100
    \quad \quad UU prend la valeur U×0,95+4U\times 0,95+4
    \quad \quad NN prend la valeur N+1N+1
    \quad Fin du Tant que
    Sortie :
    \quad Afficher NN

  2. wn+1=vn+180=0,95vn+480=0,95vn76w_{n+1} = v_{n+1} - 80 = 0,95 v_{n}+4 - 80 = 0,95 v_{n} - 76

    Or wn=vn80w_{n}=v_{n} - 80 donc vn=wn+80v_{n}=w_{n}+80

    Par conséquent :

    wn+1=0,95(wn+80)76=0,95wn+7676=0,95wnw_{n+1}= 0,95\left(w_{n}+80\right) - 76 = 0,95w_{n}+76 - 76 = 0,95 w_{n}

    On a wn+1=0,95×vnw_{n+1}= 0,95\times v_{n} a ce qui montre que la suite wnw_{n} est une suite géométrique de raison 0,950,95.

    Par ailleurs, w0=v080=4280=38 w_{0} = v_{0} - 80 = 42 - 80 = - 38, donc le premier terme de la suite (wn)\left(w_{n}\right) est 38 - 38.

    1. La raison de la suite (wn)\left(w_{n}\right) est strictement comprise entre 00 et 11 donc limn+wn=0\lim\limits_{n \rightarrow +\infty } w_{n}=0.

    2. Pour tout entier nn, vn=wn+80v_{n} = w_{n} + 80 donc limn+vn=80\lim\limits_{n \rightarrow +\infty } v_{n}=80.

    3. Au fil du temps, le nombre d'ouvrages se rapprochera de 8000080 000 sans jamais dépasser ce nombre.

      (En particulier, il n'atteindra jamais les 100000100 000 et l'algorithme ci-dessus bouclera sans fin).